Why does a meromorphic function in the (extended) complex plane have finitely many poles?












15














Let $f$ be meromorphic in $mathbb{C} cup {infty}$. Why must $f$ have only finitely many poles?



Edit: Renamed question following the comments.










share|cite|improve this question




















  • 10




    The title of the post asks a different question than the body and the question in the title is inaccurate: a meromorphic function in the complex plane MAY have infinitely many poles. The space $mathbb Ccup{infty}$ is usually called the extended complex plane or the Riemann sphere.
    – Did
    Oct 5 '11 at 9:12






  • 1




    As an explicit counterexample to the title question, consider $frac{1}{2 - e^z}$.
    – Qiaochu Yuan
    Oct 5 '11 at 15:22
















15














Let $f$ be meromorphic in $mathbb{C} cup {infty}$. Why must $f$ have only finitely many poles?



Edit: Renamed question following the comments.










share|cite|improve this question




















  • 10




    The title of the post asks a different question than the body and the question in the title is inaccurate: a meromorphic function in the complex plane MAY have infinitely many poles. The space $mathbb Ccup{infty}$ is usually called the extended complex plane or the Riemann sphere.
    – Did
    Oct 5 '11 at 9:12






  • 1




    As an explicit counterexample to the title question, consider $frac{1}{2 - e^z}$.
    – Qiaochu Yuan
    Oct 5 '11 at 15:22














15












15








15


3





Let $f$ be meromorphic in $mathbb{C} cup {infty}$. Why must $f$ have only finitely many poles?



Edit: Renamed question following the comments.










share|cite|improve this question















Let $f$ be meromorphic in $mathbb{C} cup {infty}$. Why must $f$ have only finitely many poles?



Edit: Renamed question following the comments.







complex-analysis






share|cite|improve this question















share|cite|improve this question













share|cite|improve this question




share|cite|improve this question








edited Oct 8 '11 at 6:20

























asked Oct 5 '11 at 3:44









user1205

14828




14828








  • 10




    The title of the post asks a different question than the body and the question in the title is inaccurate: a meromorphic function in the complex plane MAY have infinitely many poles. The space $mathbb Ccup{infty}$ is usually called the extended complex plane or the Riemann sphere.
    – Did
    Oct 5 '11 at 9:12






  • 1




    As an explicit counterexample to the title question, consider $frac{1}{2 - e^z}$.
    – Qiaochu Yuan
    Oct 5 '11 at 15:22














  • 10




    The title of the post asks a different question than the body and the question in the title is inaccurate: a meromorphic function in the complex plane MAY have infinitely many poles. The space $mathbb Ccup{infty}$ is usually called the extended complex plane or the Riemann sphere.
    – Did
    Oct 5 '11 at 9:12






  • 1




    As an explicit counterexample to the title question, consider $frac{1}{2 - e^z}$.
    – Qiaochu Yuan
    Oct 5 '11 at 15:22








10




10




The title of the post asks a different question than the body and the question in the title is inaccurate: a meromorphic function in the complex plane MAY have infinitely many poles. The space $mathbb Ccup{infty}$ is usually called the extended complex plane or the Riemann sphere.
– Did
Oct 5 '11 at 9:12




The title of the post asks a different question than the body and the question in the title is inaccurate: a meromorphic function in the complex plane MAY have infinitely many poles. The space $mathbb Ccup{infty}$ is usually called the extended complex plane or the Riemann sphere.
– Did
Oct 5 '11 at 9:12




1




1




As an explicit counterexample to the title question, consider $frac{1}{2 - e^z}$.
– Qiaochu Yuan
Oct 5 '11 at 15:22




As an explicit counterexample to the title question, consider $frac{1}{2 - e^z}$.
– Qiaochu Yuan
Oct 5 '11 at 15:22










2 Answers
2






active

oldest

votes


















33














By definition a pole is an isolated singularity, and so each pole has a neighborhood containing no other poles besides itself. Thus the set of all poles is discrete.



The set of poles is also closed, since its complement, the set of points at which $f$ is holomorphic, is open. (Any point where $f$ is holomorphic has a neighborhood restricted to which $f$ is holomorphic.)



Since $mathbb C cup {infty}$ is compact, any discrete and closed subset is discrete and compact, hence finite.



(Note that the reasoning of the first two paragraphs applies to a meromorphic function on any open subset of the Riemann sphere. E.g. a meromorphic function on $mathbb C$ can have infinitely many poles, but they must form a closed and discrete subset of $mathbb C$, and hence there can only be finitely many in any given bounded subset, i.e. they must accumulate at $infty$. A typical example is given by $f(z) = 1/sin z$.)






share|cite|improve this answer























  • As an exercise, extend this example and reasoning to show all nonconstant holomorphic maps between compact complex curves are finite (ie all fibers are finite sets).
    – AnonymousCoward
    Oct 5 '11 at 4:27












  • I guess this is if you are thinking of meromorphic functions as holomorphic maps to the Riemann sphere.
    – AnonymousCoward
    Oct 5 '11 at 4:30










  • here there is a proof in the first paragraph: math.wisc.edu/~dummit/sets/110b-1s.pdf
    – wqr
    Mar 29 '13 at 3:30



















1














Poles are isolated singularities, therefore they are at most countable. (There exists an injective map between the isolated singularities and $mathbb{Q}^2$)



A meromorphic function $f$ in the extended complex plane either has pole at infinity or is holomorphic at infinity. In either case there is a radius $R$ large enough so that in the region $|z|>R$ there is at most one pole, which is at infinity.



We now consider the other poles which can only be in set $K={|z|leq R}$. Suppose there are countably infinite of them, then they form a bounded sequence in $mathbb{R}^2$. By Bolzano-Weierstrass Theorem, there must be a convergent subsequence, which contradicts the fact that all poles are isolated singularities. So there can only be finite of them.






share|cite|improve this answer























    Your Answer





    StackExchange.ifUsing("editor", function () {
    return StackExchange.using("mathjaxEditing", function () {
    StackExchange.MarkdownEditor.creationCallbacks.add(function (editor, postfix) {
    StackExchange.mathjaxEditing.prepareWmdForMathJax(editor, postfix, [["$", "$"], ["\\(","\\)"]]);
    });
    });
    }, "mathjax-editing");

    StackExchange.ready(function() {
    var channelOptions = {
    tags: "".split(" "),
    id: "69"
    };
    initTagRenderer("".split(" "), "".split(" "), channelOptions);

    StackExchange.using("externalEditor", function() {
    // Have to fire editor after snippets, if snippets enabled
    if (StackExchange.settings.snippets.snippetsEnabled) {
    StackExchange.using("snippets", function() {
    createEditor();
    });
    }
    else {
    createEditor();
    }
    });

    function createEditor() {
    StackExchange.prepareEditor({
    heartbeatType: 'answer',
    autoActivateHeartbeat: false,
    convertImagesToLinks: true,
    noModals: true,
    showLowRepImageUploadWarning: true,
    reputationToPostImages: 10,
    bindNavPrevention: true,
    postfix: "",
    imageUploader: {
    brandingHtml: "Powered by u003ca class="icon-imgur-white" href="https://imgur.com/"u003eu003c/au003e",
    contentPolicyHtml: "User contributions licensed under u003ca href="https://creativecommons.org/licenses/by-sa/3.0/"u003ecc by-sa 3.0 with attribution requiredu003c/au003e u003ca href="https://stackoverflow.com/legal/content-policy"u003e(content policy)u003c/au003e",
    allowUrls: true
    },
    noCode: true, onDemand: true,
    discardSelector: ".discard-answer"
    ,immediatelyShowMarkdownHelp:true
    });


    }
    });














    draft saved

    draft discarded


















    StackExchange.ready(
    function () {
    StackExchange.openid.initPostLogin('.new-post-login', 'https%3a%2f%2fmath.stackexchange.com%2fquestions%2f69964%2fwhy-does-a-meromorphic-function-in-the-extended-complex-plane-have-finitely-ma%23new-answer', 'question_page');
    }
    );

    Post as a guest















    Required, but never shown

























    2 Answers
    2






    active

    oldest

    votes








    2 Answers
    2






    active

    oldest

    votes









    active

    oldest

    votes






    active

    oldest

    votes









    33














    By definition a pole is an isolated singularity, and so each pole has a neighborhood containing no other poles besides itself. Thus the set of all poles is discrete.



    The set of poles is also closed, since its complement, the set of points at which $f$ is holomorphic, is open. (Any point where $f$ is holomorphic has a neighborhood restricted to which $f$ is holomorphic.)



    Since $mathbb C cup {infty}$ is compact, any discrete and closed subset is discrete and compact, hence finite.



    (Note that the reasoning of the first two paragraphs applies to a meromorphic function on any open subset of the Riemann sphere. E.g. a meromorphic function on $mathbb C$ can have infinitely many poles, but they must form a closed and discrete subset of $mathbb C$, and hence there can only be finitely many in any given bounded subset, i.e. they must accumulate at $infty$. A typical example is given by $f(z) = 1/sin z$.)






    share|cite|improve this answer























    • As an exercise, extend this example and reasoning to show all nonconstant holomorphic maps between compact complex curves are finite (ie all fibers are finite sets).
      – AnonymousCoward
      Oct 5 '11 at 4:27












    • I guess this is if you are thinking of meromorphic functions as holomorphic maps to the Riemann sphere.
      – AnonymousCoward
      Oct 5 '11 at 4:30










    • here there is a proof in the first paragraph: math.wisc.edu/~dummit/sets/110b-1s.pdf
      – wqr
      Mar 29 '13 at 3:30
















    33














    By definition a pole is an isolated singularity, and so each pole has a neighborhood containing no other poles besides itself. Thus the set of all poles is discrete.



    The set of poles is also closed, since its complement, the set of points at which $f$ is holomorphic, is open. (Any point where $f$ is holomorphic has a neighborhood restricted to which $f$ is holomorphic.)



    Since $mathbb C cup {infty}$ is compact, any discrete and closed subset is discrete and compact, hence finite.



    (Note that the reasoning of the first two paragraphs applies to a meromorphic function on any open subset of the Riemann sphere. E.g. a meromorphic function on $mathbb C$ can have infinitely many poles, but they must form a closed and discrete subset of $mathbb C$, and hence there can only be finitely many in any given bounded subset, i.e. they must accumulate at $infty$. A typical example is given by $f(z) = 1/sin z$.)






    share|cite|improve this answer























    • As an exercise, extend this example and reasoning to show all nonconstant holomorphic maps between compact complex curves are finite (ie all fibers are finite sets).
      – AnonymousCoward
      Oct 5 '11 at 4:27












    • I guess this is if you are thinking of meromorphic functions as holomorphic maps to the Riemann sphere.
      – AnonymousCoward
      Oct 5 '11 at 4:30










    • here there is a proof in the first paragraph: math.wisc.edu/~dummit/sets/110b-1s.pdf
      – wqr
      Mar 29 '13 at 3:30














    33












    33








    33






    By definition a pole is an isolated singularity, and so each pole has a neighborhood containing no other poles besides itself. Thus the set of all poles is discrete.



    The set of poles is also closed, since its complement, the set of points at which $f$ is holomorphic, is open. (Any point where $f$ is holomorphic has a neighborhood restricted to which $f$ is holomorphic.)



    Since $mathbb C cup {infty}$ is compact, any discrete and closed subset is discrete and compact, hence finite.



    (Note that the reasoning of the first two paragraphs applies to a meromorphic function on any open subset of the Riemann sphere. E.g. a meromorphic function on $mathbb C$ can have infinitely many poles, but they must form a closed and discrete subset of $mathbb C$, and hence there can only be finitely many in any given bounded subset, i.e. they must accumulate at $infty$. A typical example is given by $f(z) = 1/sin z$.)






    share|cite|improve this answer














    By definition a pole is an isolated singularity, and so each pole has a neighborhood containing no other poles besides itself. Thus the set of all poles is discrete.



    The set of poles is also closed, since its complement, the set of points at which $f$ is holomorphic, is open. (Any point where $f$ is holomorphic has a neighborhood restricted to which $f$ is holomorphic.)



    Since $mathbb C cup {infty}$ is compact, any discrete and closed subset is discrete and compact, hence finite.



    (Note that the reasoning of the first two paragraphs applies to a meromorphic function on any open subset of the Riemann sphere. E.g. a meromorphic function on $mathbb C$ can have infinitely many poles, but they must form a closed and discrete subset of $mathbb C$, and hence there can only be finitely many in any given bounded subset, i.e. they must accumulate at $infty$. A typical example is given by $f(z) = 1/sin z$.)







    share|cite|improve this answer














    share|cite|improve this answer



    share|cite|improve this answer








    edited Oct 5 '11 at 14:58

























    answered Oct 5 '11 at 3:55









    Matt E

    104k8217385




    104k8217385












    • As an exercise, extend this example and reasoning to show all nonconstant holomorphic maps between compact complex curves are finite (ie all fibers are finite sets).
      – AnonymousCoward
      Oct 5 '11 at 4:27












    • I guess this is if you are thinking of meromorphic functions as holomorphic maps to the Riemann sphere.
      – AnonymousCoward
      Oct 5 '11 at 4:30










    • here there is a proof in the first paragraph: math.wisc.edu/~dummit/sets/110b-1s.pdf
      – wqr
      Mar 29 '13 at 3:30


















    • As an exercise, extend this example and reasoning to show all nonconstant holomorphic maps between compact complex curves are finite (ie all fibers are finite sets).
      – AnonymousCoward
      Oct 5 '11 at 4:27












    • I guess this is if you are thinking of meromorphic functions as holomorphic maps to the Riemann sphere.
      – AnonymousCoward
      Oct 5 '11 at 4:30










    • here there is a proof in the first paragraph: math.wisc.edu/~dummit/sets/110b-1s.pdf
      – wqr
      Mar 29 '13 at 3:30
















    As an exercise, extend this example and reasoning to show all nonconstant holomorphic maps between compact complex curves are finite (ie all fibers are finite sets).
    – AnonymousCoward
    Oct 5 '11 at 4:27






    As an exercise, extend this example and reasoning to show all nonconstant holomorphic maps between compact complex curves are finite (ie all fibers are finite sets).
    – AnonymousCoward
    Oct 5 '11 at 4:27














    I guess this is if you are thinking of meromorphic functions as holomorphic maps to the Riemann sphere.
    – AnonymousCoward
    Oct 5 '11 at 4:30




    I guess this is if you are thinking of meromorphic functions as holomorphic maps to the Riemann sphere.
    – AnonymousCoward
    Oct 5 '11 at 4:30












    here there is a proof in the first paragraph: math.wisc.edu/~dummit/sets/110b-1s.pdf
    – wqr
    Mar 29 '13 at 3:30




    here there is a proof in the first paragraph: math.wisc.edu/~dummit/sets/110b-1s.pdf
    – wqr
    Mar 29 '13 at 3:30











    1














    Poles are isolated singularities, therefore they are at most countable. (There exists an injective map between the isolated singularities and $mathbb{Q}^2$)



    A meromorphic function $f$ in the extended complex plane either has pole at infinity or is holomorphic at infinity. In either case there is a radius $R$ large enough so that in the region $|z|>R$ there is at most one pole, which is at infinity.



    We now consider the other poles which can only be in set $K={|z|leq R}$. Suppose there are countably infinite of them, then they form a bounded sequence in $mathbb{R}^2$. By Bolzano-Weierstrass Theorem, there must be a convergent subsequence, which contradicts the fact that all poles are isolated singularities. So there can only be finite of them.






    share|cite|improve this answer




























      1














      Poles are isolated singularities, therefore they are at most countable. (There exists an injective map between the isolated singularities and $mathbb{Q}^2$)



      A meromorphic function $f$ in the extended complex plane either has pole at infinity or is holomorphic at infinity. In either case there is a radius $R$ large enough so that in the region $|z|>R$ there is at most one pole, which is at infinity.



      We now consider the other poles which can only be in set $K={|z|leq R}$. Suppose there are countably infinite of them, then they form a bounded sequence in $mathbb{R}^2$. By Bolzano-Weierstrass Theorem, there must be a convergent subsequence, which contradicts the fact that all poles are isolated singularities. So there can only be finite of them.






      share|cite|improve this answer


























        1












        1








        1






        Poles are isolated singularities, therefore they are at most countable. (There exists an injective map between the isolated singularities and $mathbb{Q}^2$)



        A meromorphic function $f$ in the extended complex plane either has pole at infinity or is holomorphic at infinity. In either case there is a radius $R$ large enough so that in the region $|z|>R$ there is at most one pole, which is at infinity.



        We now consider the other poles which can only be in set $K={|z|leq R}$. Suppose there are countably infinite of them, then they form a bounded sequence in $mathbb{R}^2$. By Bolzano-Weierstrass Theorem, there must be a convergent subsequence, which contradicts the fact that all poles are isolated singularities. So there can only be finite of them.






        share|cite|improve this answer














        Poles are isolated singularities, therefore they are at most countable. (There exists an injective map between the isolated singularities and $mathbb{Q}^2$)



        A meromorphic function $f$ in the extended complex plane either has pole at infinity or is holomorphic at infinity. In either case there is a radius $R$ large enough so that in the region $|z|>R$ there is at most one pole, which is at infinity.



        We now consider the other poles which can only be in set $K={|z|leq R}$. Suppose there are countably infinite of them, then they form a bounded sequence in $mathbb{R}^2$. By Bolzano-Weierstrass Theorem, there must be a convergent subsequence, which contradicts the fact that all poles are isolated singularities. So there can only be finite of them.







        share|cite|improve this answer














        share|cite|improve this answer



        share|cite|improve this answer








        edited Nov 21 '18 at 3:21

























        answered Nov 21 '18 at 3:14









        Tian He

        277




        277






























            draft saved

            draft discarded




















































            Thanks for contributing an answer to Mathematics Stack Exchange!


            • Please be sure to answer the question. Provide details and share your research!

            But avoid



            • Asking for help, clarification, or responding to other answers.

            • Making statements based on opinion; back them up with references or personal experience.


            Use MathJax to format equations. MathJax reference.


            To learn more, see our tips on writing great answers.





            Some of your past answers have not been well-received, and you're in danger of being blocked from answering.


            Please pay close attention to the following guidance:


            • Please be sure to answer the question. Provide details and share your research!

            But avoid



            • Asking for help, clarification, or responding to other answers.

            • Making statements based on opinion; back them up with references or personal experience.


            To learn more, see our tips on writing great answers.




            draft saved


            draft discarded














            StackExchange.ready(
            function () {
            StackExchange.openid.initPostLogin('.new-post-login', 'https%3a%2f%2fmath.stackexchange.com%2fquestions%2f69964%2fwhy-does-a-meromorphic-function-in-the-extended-complex-plane-have-finitely-ma%23new-answer', 'question_page');
            }
            );

            Post as a guest















            Required, but never shown





















































            Required, but never shown














            Required, but never shown












            Required, but never shown







            Required, but never shown

































            Required, but never shown














            Required, but never shown












            Required, but never shown







            Required, but never shown







            Popular posts from this blog

            Can a sorcerer learn a 5th-level spell early by creating spell slots using the Font of Magic feature?

            Does disintegrating a polymorphed enemy still kill it after the 2018 errata?

            A Topological Invariant for $pi_3(U(n))$